maternity

Réussis tes devoirs et examens dès maintenant avec Quizwiz!

A nurse is caring for a client scheduled for a cesarean delivery. The nurse reviews the client's health record, knowing that which finding needs to be further investigated before delivery?

White blood cell count of 35,000 mm3

A nurse is caring for a client in labor. The nurse notes the presence of fetal bradycardia on the fetal monitor and suspects that the umbilical cord is compressed. The nurse immediately places the client in what position?

With the hips elevated

The nurse is assigned to care for a client who is in early labor. When collecting data from the client, it is most important for the nurse to first determine which of the following?

Baseline fetal heart rate

A client in labor asks the nurse why it is so important to void frequently during labor. The nurse responds, using knowledge that the important reason is to:

Ensure labor progress and prevent injury.

The mother asks how much her baby will sleep every day. The best response is: a. "He will only wake up to be fed, and you should not bother him between feedings." b. "A newborn sleeps about 17 hours a day with periods of wakefulness gradually increasing." c. "He will probably follow your same sleep and wake patterns, so you can expect him to be awake soon." d. "You should try to limit day sleep to about 3 hours so he will sleep through the night."

"A newborn sleeps about 17 hours a day with periods of wakefulness gradually increasing."

A woman who is 14 weeks pregnant says that she has abstained from alcohol during her first trimester and would like to know if it is now safe to have a drink with dinner. The health-care worker's best response would be: a. "As you are in your second trimester, there is no problem with having one drink with dinner." b. "One drink every night is too much. One drink three times a week should be fine." c. "Because no one knows how much or how little alcohol it takes to cause fetal problems, it is recommended that you abstain throughout your pregnancy." d. "Because you are in your second trimester, you can drink as much as you like."

"Because no one knows how much or how little alcohol it takes to cause fetal problems, it is recommended that you abstain throughout your pregnancy."

A Chinese woman rejects the plan for her to bring the newborn to the health clinic before the seventh day of life to repeat the test for phenylketonuria (PKU) stating that it is too early for her to come. The culturally competent health-care worker would ask: a. "Do you lack transportation to the clinic?" b. "Isn't your baby's health important enough to come to the clinic?" c. "Do you realize that the state requires this test to be done?" d. "Do you need more time to heal after your delivery?"

"Do you need more time to heal after your delivery?"

During a prenatal visit, the nurse is explaining dietary management to a client with diabetes mellitus. The nurse determines that the teaching has been effective when the client states:

"I need to increase the fiber in my diet to control my blood glucose and prevent constipation."

A client is pregnant, has a history of heart disease, and has been instructed on care at home. Which statement by the client would indicate that the client understands her needs?

"I should avoid stressful situations."

A client who is pregnant has been instructed on prevention of genital tract infections. Which statement by the client indicates an understanding of these prevention measures?

"I should choose underwear with a cotton panel liner."

A woman delivered a baby 1 hour ago. The nurse is ready to assist her with breastfeeding. The woman states she cannot breastfeed until her "good" milk comes in. She says she is afraid the "thin" milk will harm the baby. The nurse's best response is: a. "You should begin breastfeeding now or your milk will never come in." b. "I understand that you believe this colostrum is unhealthy for the baby. Let's talk about feeding alternatives until your milk comes in." c. "If you don't provide your baby with the benefits of colostrum, he will have more childhood illnesses than normal." d. "Your baby will not learn to suck correctly if you don't start breastfeeding now."

"I understand that you believe this colostrum is unhealthy for the baby. Let's talk about feeding alternatives until your milk comes in."

A perinatal client with a history of heart disease has been instructed on care at home. Which of the following statements if made by the client would indicate the need for further instructions?

"It is best to rest on my right side."

A nursing student is conducting a clinical conference regarding the hormones that are related to pregnancy, and the instructor asks the student about the function of progesterone. Which of the following responses, if made by the student, indicates an understanding of the function of this hormone?

"It maintains the uterine lining for implantation and relaxes all smooth muscle, including the uterus."

Parents of a newborn ask about the baby's vision and specifically want to know what visual stimuli they should provide for the newborn. The most appropriate response is: a. "Babies can see very little until about age 3 months." b. "It is important to shield the baby's eyes. You should ask your physician what visual stimulation to offer your child." c. "The baby's eyes are immature and must be protected. Any visual stimuli should not contain bright or strong colors." d. "Newborns have good near vision. A brightly colored mobile to hang over the crib is a good choice for your baby."

"Newborns have good near vision. A brightly colored mobile to hang over the crib is a good choice for your baby."

The nursing instructor asks the nursing student about the physiology related to the cessation of ovulation that occurs during pregnancy. Which response by the student indicates an understanding of this physiological process?

"Ovulation ceases during pregnancy because the circulating levels of estrogen and progesterone are high."

A pregnant anemic client is concerned about her baby's condition following delivery. Which nursing response would best support the client?

"The effects of anemia on your baby are difficult to predict, but let's review your plan of care to ensure you are providing the best nutrition and growth potential."

A nurse is assisting in performing Leopold's maneuvers. When the client asks what these are for, the nurse's best response is that these maneuvers help to determine:

Fetal position

A new mother of a baby girl who has a 2 1/2-year-old son at home says, "I don't know how I'll ever manage both children when I get home." The most informative response would be: a. "Your son is a big brother now and will love his new sister." b. "You might consider letting your son stay with his grandparents for several weeks to allow him to adjust to the new baby." c. "Make sure your husband bonds with your son while you are busy with your new daughter." d. "You may see babyish behavior in your son, which is a typical reaction."

"You may see babyish behavior in your son, which is a typical reaction."

A nurse is collecting data on a client with severe preeclampsia. Choose the findings that would be noted in severe preeclampsia. Select all that apply.

-Oliguria. -Proteinuria 3+ -Blood pressure 168/116 mm Hg

A client asks, "What does it mean that the baby is at minus one?" The nurse should explain to the client that the fetal presenting part is isolated:

1 cm above the ischial spines

A nurse is reviewing the record of a client in the labor room and notes that the nurse-midwife has documented that the fetus is at minus one station. The nurse determines that the fetal presenting part is:

1 cm above the ischial spines

A nurse is monitoring a client in labor who is receiving Pitocin and notes that the client is experiencing hypertonic uterine contractions. List in order of priority the actions that the nurse takes. 1.Stop of Pitocin infusion 2.Perform a vaginal examination 3.Reposition the client 4.Check the client's blood pressure and heart rate 5.Administer oxygen by face mask at 8 to 10 L/min

1, 4, 2. 5, 3. If uterine hypertonicity occurs, the nurse immediately would intervene to reduce uterine activity and increase fetal oxygenation. The nurse would stop the Pitocin infusion and increase the rate of the nonadditive solution, check maternal BP for hyper or hypotension, position the woman in a side-lying position, and administer oxygen by snug face mask at 8-10 L/min. The nurse then would attempt to determine the cause of the uterine hypertonicity and perform a vaginal exam to check for prolapsed cord

A nurse is performing an assessment of a client who is scheduled for a cesarean delivery. Which assessment finding would indicate a need to contact the physician? 1.Fetal heart rate of 180 beats per minute 2.White blood cell count of 12,000 3.Maternal pulse rate of 85 beats per minute 4.Hemoglobin of 11.0 g/dL

1. A normal fetal heart rate is 120-160 beats per minute. A count of 180 beats per minute could indicate fetal distress and would warrant physician notification. By full term, a normal maternal hemoglobin range is 11-13 g/dL as a result of the hemodilution caused by an increase in plasma volume during pregnancy.

A nurse is caring for a client in labor and is monitoring the fetal heart rate patterns. The nurse notes the presence of episodic accelerations on the electronic fetal monitor tracing. Which of the following actions is most appropriate? 1.Document the findings and tell the mother that the monitor indicates fetal well-being 2.Take the mothers vital signs and tell the mother that bed rest is required to conserve oxygen. 3.Notify the physician or nurse mid-wife of the findings. 4.Reposition the mother and check the monitor for changes in the fetal tracing

1. Accelerations are transient increases in the fetal heart rate that often accompany contractions or are caused by fetal movement. Episodic accelerations are thought to be a sign of fetal-well being and adequate oxygen reserve.

When monitoring the fetal heart rate of a client in labor, the nurse identifies an elevation of 15 beats above the baseline rate of 135 beats per minute lasting for 15 seconds. This should be documented as: 1.An acceleration 2.An early elevation 3.A sonographic motion 4.A tachycardic heart rate

1. An acceleration is an abrupt elevation above the baseline of 15 beats per minute for 15 seconds; if the acceleration persists for more than 10 minutes it is considered a change in baseline rate. A tachycardic FHR is above 160 beats per minute

The breathing technique that the mother should be instructed to use as the fetus' head is crowning is: 1.Blowing 2.Slow chest 3.Shallow 4.Accelerated-decelerated

1. Blowing forcefully through the mouth controls the strong urge to push and allows for a more controlled birth of the head.

A maternity nurse is caring for a client with abruptio placenta and is monitoring the client for disseminated intravascular coagulopathy. Which assessment finding is least likely to be associated with disseminated intravascular coagulation? 1.Swelling of the calf in one leg 2.Prolonged clotting times 3.Decreased platelet count 4.Petechiae, oozing from injection sites, and hematuria

1. DIC is a state of diffuse clotting in which clotting factors are consumed, leading to widespread bleeding. Platelets are decreased because they are consumed by the process; coagulation studies show no clot formation (and are thus normal to prolonged); and fibrin plugs may clog the microvasculature diffusely, rather than in an isolated area. The presence of petechiae, oozing from injection sites, and hematuria are signs associated with DIC. Swelling and pain in the calf of one leg are more likely to be associated with thrombophebitis.

A client is admitted to the birthing suite in early active labor. The priority nursing intervention on admission of this client would be: 1.Auscultating the fetal heart 2.Taking an obstetric history 3.Asking the client when she last ate 4.Ascertaining whether the membranes were ruptured

1. Determining the fetal well-being supersedes all other measures. If the FHR is absent or persistently decelerating, immediate intervention is required.

A nurse is reviewing the record of a client in the labor room and notes that the nurse midwife has documented that the fetus is at -1 station. The nurse determines that the fetal presenting part is: 1.1 cm above the ischial spine 2.1 fingerbreadth below the symphysis pubis 3.1 inch below the coccyx 4.1 inch below the iliac crest

1. Station is the relationship of the presenting part to an imaginary line drawn between the ischial spines, is measured in centimeters, and is noted as a negative number above the line and a positive number below the line. At -1 station, the fetal presenting part is 1 cm above the ischial spines.

A nurse in the labor room is performing a vaginal assessment on a pregnant client in labor. The nurse notes the presence of the umbilical cord protruding from the vagina. Which of the following would be the initial nursing action? 1.Place the client in Trendelenburg's position 2.Call the delivery room to notify the staff that the client will be transported immediately 3.Gently push the cord into the vagina 4.Find the closest telephone and stat page the physician

1. When cord prolapse occurs, prompt actions are taken to relieve cord compression and increase fetal oxygenation. The mother should be positioned with the hips higher than the head to shift the fetal presenting part toward the diaphragm. The nurse should push the call light to summon help, and other staff members should call the physician and notify the delivery room. No attempt should be made to replace the cord. The examiner, however, may place a gloved hand into the vagina and hold the presenting part off of the umbilical cord. Oxygen at 8 to 10 L/min by face mask is delivered to the mother to increase fetal oxygenation.

1.A nurse is caring for a client in labor. The nurse determines that the client is beginning in the 2nd stage of labor when which of the following assessments is noted? A.The client begins to expel clear vaginal fluid B.The contractions are regular C.The membranes have ruptured D.The cervix is dilated completely

1.4. The second stage of labor begins when the cervix is dilated completely and ends with the birth of the neonate.

A nurse is caring for a woman in labor. The nurse monitors the baseline fetal heart rate (FHR) and would document that the FHR is normal if which of the following were noted?

150 beats per minute

A nurse is caring for a client in labor who is receiving Pitocin by IV infusion to stimulate uterine contractions. Which assessment finding would indicate to the nurse that the infusion needs to be discontinued? 1.Three contractions occurring within a 10-minute period 2.A fetal heart rate of 90 beats per minute 3.Adequate resting tone of the uterus palpated between contractions 4.Increased urinary output

2. A normal fetal heart rate is 120-160 BPM. Bradycardia or late or variable decelerations indicate fetal distress and the need to discontinue to pitocin. The goal of labor augmentation is to achieve three good-quality contractions in a 10-minute period.

When examining the fetal monitor strip after rupture of the membranes in a laboring client, the nurse notes variable decelerations in the fetal heart rate. The nurse should: 1.Stop the oxytocin infusion 2.Change the client's position 3.Prepare for immediate delivery 4.Take the client's blood pressure

2. Variable decelerations usually are seen as a result of cord compression; a change of position will relieve pressure on the cord.

A nurse is caring for a client in the second stage of labor. The client is experiencing uterine contractions every 2 minutes and cries out in pain with each contraction. The nurse recognizes this behavior as: 1.Exhaustion 2.Fear of losing control 3.Involuntary grunting 4.Valsalva's maneuver

2. Pains, helplessness, panicking, and fear of losing control are possible behaviors in the 2nd stage of labor.

A client arrives at a birthing center in active labor. Her membranes are still intact, and the nurse-midwife prepares to perform an amniotomy. A nurse who is assisting the nurse-midwife explains to the client that after this procedure, she will most likely have: 1.Less pressure on her cervix 2.Increased efficiency of contractions 3.Decreased number of contractions 4.The need for increased maternal blood pressure monitoring

2. Amniotomy can be used to induce labor when the condition of the cervix is favorable (ripe) or to augment labor if the process begins to slow. Rupturing of membranes allows the fetal head to contact the cervix more directly and may increase the efficiency of contractions.

A nurse is admitting a pregnant client to the labor room and attaches an external electronic fetal monitor to the client's abdomen. After attachment of the monitor, the initial nursing assessment is which of the following? 1.Identifying the types of accelerations 2.Assessing the baseline fetal heart rate 3.Determining the frequency of the contractions 4.Determining the intensity of the contractions

2. Assessing the baseline fetal heart rate is important so that abnormal variations of the baseline rate will be identified if they occur. Options 1 and 3 are important to assess, but not as the first priority.

A nurse is beginning to care for a client in labor. The physician has prescribed an IV infusion of Pitocin. The nurse ensures that which of the following is implemented before initiating the infusion? 1.Placing the client on complete bed rest 2.Continuous electronic fetal monitoring 3.An IV infusion of antibiotics 4.Placing a code cart at the client's bedside

2. Continuous electronic fetal monitoring should be implemented during an IV infusion of Pitocin.

A nurse explains the purpose of effleurage to a client in early labor. The nurse tells the client that effleurage is: 1.A form of biofeedback to enhance bearing down efforts during delivery 2.Light stroking of the abdomen to facilitate relaxation during labor and provide tactile stimulation to the fetus 3.The application of pressure to the sacrum to relieve a backache 4.Performed to stimulate uterine activity by contracting a specific muscle group while other parts of the body rest

2. Effleurage is a specific type of cutaneous stimulation involving light stroking of the abdomen and is used before transition to promote relaxation and relieve mild to moderate pain. Effleurage provides tactile stimulation to the fetus.

A nurse in a labor room is assisting with the vaginal delivery of a newborn infant. The nurse would monitor the client closely for the risk of uterine rupture if which of the following occurred? 1.Hypotonic contractions 2.Forceps delivery 3.Schultz delivery 4.Weak bearing down efforts

2. Excessive fundal pressure, forceps delivery, violent bearing down efforts, tumultuous labor, and shoulder dystocia can place a woman at risk for traumatic uterine rupture. Hypotonic contractions and weak bearing down efforts do not alone add to the risk of rupture because they do not add to the stress on the uterine wall.

A nurse in the labor room is preparing to care for a client with hypertonic uterine dysfunction. The nurse is told that the client is experiencing uncoordinated contractions that are erratic in their frequency, duration, and intensity. The priority nursing intervention would be to: 1.Monitor the Pitocin infusion closely 2.Provide pain relief measures 3.Prepare the client for an amniotomy 4.Promote ambulation every 30 minutes

2. Management of hypertonic labor depends on the cause. Relief of pain is the primary intervention to promote a normal labor pattern.

A nurse is monitoring a client in labor. The nurse suspects umbilical cord compression if which of the following is noted on the external monitor tracing during a contraction? 1.Early decelerations 2.Variable decelerations 3.Late decelerations 4.Short-term variability

2. Variable decelerations occur if the umbilical cord becomes compressed, thus reducing blood flow between the placenta and the fetus. Early decelerations result from pressure on the fetal head during a contraction. Late decelerations are an ominous pattern in labor because it suggests uteroplacental insufficiency during a contraction. Short-term variability refers to the beat-to-beat range in the fetal heart rate.

An ultrasound is performed on a client at term gestation that is experiencing moderate vaginal bleeding. The results of the ultrasound indicate that an abruptio placenta is present. Based on these findings, the nurse would prepare the client for: 1.Complete bed rest for the remainder of the pregnancy 2.Delivery of the fetus 3.Strict monitoring of intake and output 4.The need for weekly monitoring of coagulation studies until the time of delivery

2. The goal of management in abruptio placentae is to control the hemorrhage and deliver the fetus as soon as possible. Delivery is the treatment of choice if the fetus is at term gestation or if the bleeding is moderate to severe and the mother or fetus is in jeopardy.

During the period of induction of labor, a client should be observed carefully for signs of: 1.Severe pain 2.Uterine tetany 3.Hypoglycemia 4.Umbilical cord prolapse

2. Uterine tetany could result from the use of oxytocin to induce labor. Because oxytocin promotes powerful uterine contractions, uterine tetany may occur. The oxytocin infusion must be stopped to prevent uterine rupture and fetal compromise.

The Moro (startle) reflex, which is present at birth, disappears normally at the age of: a. 3 months. b. 4 months. c. 5 months. d. 6 months.

3 months.

A client who is gravida 1, para 0 is admitted in labor. Her cervix is 100% effaced, and she is dilated to 3 cm. Her fetus is at +1 station. The nurse is aware that the fetus' head is: 1.Not yet engaged 2.Entering the pelvic inlet 3.Below the ischial spines 4.Visible at the vaginal opening

3. A station of +1 indicates that the fetal head is 1 cm below the ischial spines.

After doing Leopold's maneuvers, the nurse determines that the fetus is in the ROP position. To best auscultate the fetal heart tones, the Doppler is placed: 1.Above the umbilicus at the midline 2.Above the umbilicus on the left side 3.Below the umbilicus on the right side 4.Below the umbilicus near the left groin

3. Fetal heart tones are best auscultated through the fetal back; because the position is ROP (right occiput presenting), the back would be below the umbilicus and on the right side.

A maternity nurse is preparing to care for a pregnant client in labor who will be delivering twins. The nurse monitors the fetal heart rates by placing the external fetal monitor: 1.Over the fetus that is most anterior to the mothers abdomen 2.Over the fetus that is most posterior to the mothers abdomen 3.So that each fetal heart rate is monitored separately 4.So that one fetus is monitored for a 15-minute period followed by a 15 minute fetal monitoring period for the second fetus

3. In a client with a multi-fetal pregnancy, each fetal heart rate is monitored separately.

A nurse is assessing a pregnant client in the 2nd trimester of pregnancy who was admitted to the maternity unit with a suspected diagnosis of abruptio placentae. Which of the following assessment findings would the nurse expect to note if this condition is present? 1.Absence of abdominal pain 2.A soft abdomen 3.Uterine tenderness/pain 4.Painless, bright red vaginal bleeding

3. In abruptio placentae, acute abdominal pain is present. Uterine tenderness and pain accompanies placental abruption, especially with a central abruption and trapped blood behind the placenta. The abdomen will feel hard and boardlike on palpation as the blood penetrates the myometrium and causes uterine irritability. Observation of the fetal monitoring often reveals increased uterine resting tone, caused by failure of the uterus to relax in attempt to constrict blood vessels and control bleeding.

A nurse in the labor room is caring for a client in the active phases of labor. The nurse is assessing the fetal patterns and notes a late deceleration on the monitor strip. The most appropriate nursing action is to: 1.Place the mother in the supine position 2.Document the findings and continue to monitor the fetal patterns 3.Administer oxygen via face mask 4.Increase the rate of pitocin IV infusion

3. Late decelerations are due to uteroplacental insufficiency as the result of decreased blood flow and oxygen to the fetus during the uterine contractions. This causes hypoxemia; therefore oxygen is necessary. The supine position is avoided because it decreases uterine blood flow to the fetus. The client should be turned to her side to displace pressure of the gravid uterus on the inferior vena cava. An intravenous pitocin infusion is discontinued when a late deceleration is noted.

A maternity nurse is preparing for the admission of a client in the 3rd trimester of pregnancy that is experiencing vaginal bleeding and has a suspected diagnosis of placenta previa. The nurse reviews the physician's orders and would question which order? 1.Prepare the client for an ultrasound 2.Obtain equipment for external electronic fetal heart monitoring 3.Obtain equipment for a manual pelvic examination 4.Prepare to draw a Hgb and Hct blood sample

3. Manual pelvic examinations are contraindicated when vaginal bleeding is apparent in the 3rd trimester until a diagnosis is made and placental previa is ruled out. Digital examination of the cervix can lead to maternal and fetal hemorrhage. A diagnosis of placenta previa is made by ultrasound. The H/H levels are monitored, and external electronic fetal heart rate monitoring is initiated. External fetal monitoring is crucial in evaluating the fetus that is at risk for severe hypoxia

A nurse is developing a plan of care for a client experiencing dystocia and includes several nursing interventions in the plan of care. The nurse prioritizes the plan of care and selects which of the following nursing interventions as the highest priority? 1.Keeping the significant other informed of the progress of the labor 2.Providing comfort measures 3.Monitoring fetal heart rate 4.Changing the client's position frequently

3. The priority is to monitor the fetal heart rate.

A nurse is assigned to care for a client with hypotonic uterine dysfunction and signs of a slowing labor. The nurse is reviewing the physician's orders and would expect to note which of the following prescribed treatments for this condition? 1.Medication that will provide sedation 2.Increased hydration 3.Oxytocin (Pitocin) infusion 4.Administration of a tocolytic medication

3. Therapeutic management for hypotonic uterine dysfunction includes oxytocin augmentation and amniotomy to stimulate a labor that slows.

The physician asks the nurse the frequency of a laboring client's contractions. The nurse assesses the client's contractions by timing from the beginning of one contraction: 1.Until the time it is completely over 2.To the end of a second contraction 3.To the beginning of the next contraction 4.Until the time that the uterus becomes very firm

3. This is the way to determine the frequency of the contractions

The nurse observes the client's amniotic fluid and decides that it appears normal, because it is: 1.Clear and dark amber in color 2.Milky, greenish yellow, containing shreds of mucus 3.Clear, almost colorless, and containing little white specks 4.Cloudy, greenish-yellow, and containing little white specks

3. by 36 weeks' gestation, normal amniotic fluid is colorless with small particles of vernix caseosa present.

The young pregnant woman with type 2 diabetes mellitus should get at least ____ minutes of exercise a day to control weight and blood glucose. a. 10 b. 15 c. 30 d. 60

30

A nurse is assisting in performing an assessment on a client who is at 32 weeks of gestation. The nurse measures the fundal height in centimeters and expects the findings to be which of the following?

32 cm

A nurse is monitoring a client in active labor and notes that the client is having contractions every 3 minutes that last 45 seconds. The nurse notes that the fetal heart rate between contractions is 100 BPM. Which of the following nursing actions is most appropriate? 1.Encourage the client's coach to continue to encourage breathing exercises 2.Encourage the client to continue pushing with each contraction 3.Continue monitoring the fetal heart rate 4.Notify the physician or nurse mid-wife

4. A normal fetal heart rate is 120-160 beats per minute. Fetal bradycardia between contractions may indicate the need for immediate medical management, and the physician or nurse mid-wife needs to be notified.

A laboring client complains of low back pain. The nurse replies that this pain occurs most when the position of the fetus is: 1.Breech 2.Transverse 3.Occiput anterior 4.Occiput posterior

4. A persistent occiput-posterior position causes intense back pain because of fetal compression of the sacral nerves. Occiput anterior is the most common fetal position and does not cause back pain.

At 38 weeks' gestation, a client is having late decelerations. The fetal pulse oximeter shows 75% to 85%. The nurse should: 1.Discontinue the catheter, if the reading is not above 80% 2.Discontinue the catheter, if the reading does not go below 30% 3.Advance the catheter until the reading is above 90% and continue monitoring 4.Reposition the catheter, recheck the reading, and if it is 55%, keep monitoring

4. Adjusting the catheter would be indicated. Normal fetal pulse oximetry should be between 30% and 70%. 75% to 85% would indicate maternal readings.

A pregnant client is admitted to the labor room. An assessment is performed, and the nurse notes that the client's hemoglobin and hematocrit levels are low, indicating anemia. The nurse determines that the client is at risk for which of the following? 1.A loud mouth 2.Low self-esteem 3.Hemorrhage 4.Postpartum infections

4. Anemic women have a greater likelihood of cardiac decompensation during labor, postpartum infection, and poor wound healing. Anemia does not specifically present a risk for hemorrhage. Having a loud mouth is only related to the person typing up this test.

A nurse assists in the vaginal delivery of a newborn infant. After the delivery, the nurse observes the umbilical cord lengthen and a spurt of blood from the vagina. The nurse documents these observations as signs of: 1.Hematoma 2.Placenta previa 3.Uterine atony 4.Placental separation

4. As the placenta separates, it settles downward into the lower uterine segment. The umbilical cord lengthens, and a sudden trickle or spurt of blood appears.

A nurse in the postpartum unit is caring for a client who has just delivered a newborn infant following a pregnancy with placenta previa. The nurse reviews the plan of care and prepares to monitor the client for which of the following risks associated with placenta previa? 1.Disseminated intravascular coagulation 2.Chronic hypertension 3.Infection 4.Hemorrhage

4. Because the placenta is implanted in the lower uterine segment, which does not contain the same intertwining musculature as the fundus of the uterus, this site is more prone to bleeding.

A nurse in the delivery room is assisting with the delivery of a newborn infant. After the delivery of the newborn, the nurse assists in delivering the placenta. Which observation would indicate that the placenta has separated from the uterine wall and is ready for delivery? 1.The umbilical cord shortens in length and changes in color 2.A soft and boggy uterus 3.Maternal complaints of severe uterine cramping 4.Changes in the shape of the uterus

4. Signs of placental separation include lengthening of the umbilical cord, a sudden gush of dark blood from the introitus (vagina), a firmly contracted uterus, and the uterus changing from a discoid (like a disk) to a globular (like a globe) shape. The client may experience vaginal fullness, but not severe uterine cramping. I am going to look more into this answer. According to our book on page 584, this is not one of our options.

. A nurse is caring for a client in labor and prepares to auscultate the fetal heart rate by using a Doppler ultrasound device. The nurse most accurately determines that the fetal heart sounds are heard by: 1.Noting if the heart rate is greater than 140 BPM 2.Placing the diaphragm of the Doppler on the mother abdomen 3.Performing Leopold's maneuvers first to determine the location of the fetal heart 4.Palpating the maternal radial pulse while listening to the fetal heart rate

4. The nurse simultaneously should palpate the maternal radial or carotid pulse and auscultate the fetal heart rate to differentiate the two. If the fetal and maternal heart rates are similar, the nurse may mistake the maternal heart rate for the fetal heart rate. Leopold's maneuvers may help the examiner locate the position of the fetus but will not ensure a distinction between the two rates.

A client in labor is transported to the delivery room and is prepared for a cesarean delivery. The client is transferred to the delivery room table, and the nurse places the client in the: 1.Trendelenburg's position with the legs in stirrups 2.Semi-Fowler position with a pillow under the knees 3.Prone position with the legs separated and elevated 4.Supine position with a wedge under the right hip

4. Vena cava and descending aorta compression by the pregnant uterus impedes blood return from the lower trunk and extremities. This leads to decreasing cardiac return, cardiac output, and blood flow to the uterus and the fetus. The best position to prevent this would be side-lying with the uterus displaced off of abdominal vessels. Positioning for abdominal surgery necessitates a supine position; however, a wedge placed under the right hip provides displacement of the uterus.

A client delivers a viable male neonate who is given APGAR scores of 8 and 9 at 1 and 5 minutes. The nurse determines the physical condition of the neonate to be:

Good

A client tells the nurse her contractions are getting stronger and that she is getting tired. She appears restless, asks the nurse not to leave her alone, and states, "I can't take it anymore." Considering the client's behavior, the nurse suspects she is dilated:

8 to 10 cm

The term fetus refers to an embryo that is _____ weeks old. a. 2 b. 3 c. 6 d. 9

9

A nurse is providing instructions to a pregnant client with genital herpes about the measures that need to be implemented to protect the fetus. The nurse tells the client that:

A cesarean section will be necessary if vaginal lesions are present at the time of labor.

The nurse is assigned to work in the delivery room and is assisting with caring for a client who has just delivered a newborn infant. The nurse is monitoring for signs of placental separation, knowing that which of the following indicates that the placenta has separated?

A change in the uterine contour

The client is admitted to the labor suite complaining of painless vaginal bleeding. The nurse assists with the examination of the client, knowing that a routine labor procedure that is contraindicated with this client's situation is:

A manual pelvic examination

A client with type 1 diabetes mellitus in the first trimester of pregnancy is scheduled for a health care provider's visit. The client asks the nurse whether a change in the medication to treat the diabetes will occur. The nurse bases the response on which of the following?

A steady increase in insulin will be needed.

A nurse is collecting data from a client and is reviewing the client's health record to determine the risk for preterm labor. Which of the following findings would place the client at this risk?

A urinary tract infection

21. During the last trimester of pregnancy, the nurse recommends that the woman wear low-heeled shoes. What is the nurse trying to prevent with this recommendation? a. Lower back pain b. Leg cramps c. Leg swelling d. Joint pain

ANS: A A remedy for backache is to wear low-heeled shoes. PTS: 1 DIF: Cognitive Level: Application REF: Page 786 OBJ: 7 TOP: Low back pain KEY: Nursing Process Step: Implementation

11. During the second stage of labor, how often should the nurse should monitor the fetal heart rate? a. Every 5 minutes b. Every 15 minutes c. Every 30 minutes d. Every hour

ANS: A Fetal heart rate should be assessed every 5 minutes during the second stage of labor.

10. Using Leopold maneuvers to assess fetal position, the nurse finds a soft rounded prominence at the level of the fundus, a hard round prominence just above the symphysis pubis, and nodulations on the left side of the uterus. How should the nurse document the fetal position? a. Right occiput anterior (ROA), vertex b. Left occiput anterior (LOA), vertex c. Right occiput transverse (ROT), breech d. Left occiput anterior (LOA), breech

ANS: A Fetal position can be determined by the Leopold maneuver, which defines the relationship of the presenting part to the maternal pelvis quadrant. A soft rounded prominence at the level of the fundus, a hard round prominence just above the symphysis pubis, and nodulations on the left side of the uterus indicate a right occiput anterior (ROA), vertex positioning.

35. The nurse assesses a reactive result to a nonstress test when the fetal heart rate increases _____ beats per minute.

ANS: 15 fifteen The reactive criterion is that the fetal heart rate will increase 15 beats per minute when stimulated in the nonstress test. PTS: 1 DIF: Cognitive Level: Application REF: Page 776 OBJ: 3 TOP: Nonstress test KEY: Nursing Process Step: Assessment

COMPLETION 34. The nurse explains to the patient whose membranes ruptured an hour ago that delivery is usually accomplished in ____ to _____ hours postrupture.

ANS: 18, 24 eighteen, twenty-four After the rupture of membranes, labor is usually accomplished in 18 to 24 hours.

34. During the 30th week of gestation, the nurse would anticipate that the fundal height would be _____ centimeters above the symphysis.

ANS: 30 thirty The fundal height is equal to the weeks of gestation. PTS: 1 DIF: Cognitive Level: Application REF: Page 773 OBJ: 3 TOP: Fundal height KEY: Nursing Process Step: Assessment

35. A primigravida has a pelvis of the android type, which usually means the delivery will be a _______________.

ANS: cesarean The narrow outlet of the android-type pelvis usually requires a cesarean delivery.

32. The nurse instructor reminds the nursing student that the "Shiny Schultz" is a name given to the _____________ side of the placenta.

ANS: fetal The fetal side of the placenta is called the Shiny Schultz and the maternal side is called the Dirty Duncan. PTS: 1 DIF: Cognitive Level: Knowledge REF: Page 764 OBJ: 1 TOP: Placental sides KEY: Nursing Process Step: Intervention

33. The chorion and the amnion are the two components of the ________ __________.

ANS: fetal membrane The fetal membrane is composed of the chorion and the amnion. PTS: 1 DIF: Cognitive Level: Knowledge REF: Page 764 OBJ: 1 TOP: Fetal membrane KEY: Nursing Process Step: N/A

36. A nurse shows the patient an x-ray of the fetal spine in parallel alignment with the mother's to demonstrate a ________ lie.

ANS: longitudinal A longitudinal lie is when the fetal spine and the maternal spine are parallel to each other.

26. When the young primigravida asks about how to adjust her diet for her pregnancy, what should the nurse suggest the mother add to her diet? a. Leafy green vegetables and fruit b. Beef and poultry c. Foods high in sodium and potassium d. Bread and grains

ANS: A A pregnant woman should eat foods containing roughage, such as raw fruits, vegetables, and cereals with bran. PTS: 1 DIF: Cognitive Level: Comprehension REF: Page 770, Table 25-1 OBJ: 6 TOP: Diet KEY: Nursing Process Step: Implementation

24. The physician has decided to induce labor with prostaglandin gel and an amniotomy. When should the nurse expect that labor will start? a. 1 hour b. 4 hours c. 8 hours d. 12 hours

ANS: A Medically approved methods of inducing labor include prostaglandin gel application that usually induces labor in 1 hour or less.

4. Why is the size and shape of the true pelvis more important than that of the false pelvis? a. The fetal head must be able to pass through the true pelvis. b. The true pelvis are the mother's measurements. c. The size of the false pelvis can change. d. The size of the true pelvis needs to be larger.

ANS: A The size and shape of the true pelvis is more important than the false pelvis because the fetal head must be able to pass through for vaginal delivery to occur.

1. A woman who is 38 weeks pregnant tells the nurse that the baby has dropped and she is having urinary frequency again. What do these symptoms describe? a. Lightening b. Braxton-Hicks contractions c. Initiation of labor d. Engagement

ANS: A The symptoms of lightening are a return of urinary frequency, and the patient is able to breathe more normally.

13. When observing the fetal heart monitor, the nurse recognizes the fetal heart rate (FHR) decreases to 120 bpm at the beginning of a contraction and returns to a baseline of 155 bpm at the end of the contraction. What should this indicate to the nurse? a. Early deceleration due to head compression b. That the fetus is in acute distress c. Variable decelerations due to cord compression d. That these are late decelerations

ANS: A This indicates early decelerations because of head compression.

30. When the nurse performs the Nitrazine test on vaginal secretions of a patient who thinks her membranes have ruptured, the paper turns yellow. What does this finding indicate? a. Acidic discharge, membranes intact b. Acidic discharge, membranes have ruptured c. Neutral, not enough discharge to measure d. Alkaline, membranes have ruptured

ANS: A When the Nitrazine paper turns yellow it is indicative of acidic discharge, meaning the membranes are intact. Amniotic fluid is alkaline and turns the paper blue.

30. The nurse concludes that the prenatal patient has no need for further instruction when she correctly states that amniocentesis can determine which of the baby's characteristics? (Select all that apply.) a. Sex b. Maturity c. Approximate weight d. Health e. Genetic defects

ANS: A, B, D, E The amniocentesis can reveal the sex, maturity, health, and some genetic defects. PTS: 1 DIF: Cognitive Level: Analysis REF: Page 776 OBJ: 3 TOP: Amniocentesis KEY: Nursing Process Step: Evaluation

31. Which of the following demonstrate culturally competent care of the pregnant patient? (Select all that apply.) a. Discuss beliefs with the patient and incorporate them in the plan of care. b. Prohibit visits from anyone other than immediate family members. c. Require the patient's participation in every aspect of the health care system. d. Maintain the patient's modesty at all times. e. Strive to maintain a harmonious environment for the patient.

ANS: A, D, E The nurse should discuss the patient's cultural beliefs and incorporate as many as possible into the plan of care. Modesty is important in almost all cultures, and the nurse should take measures to ensure the patient's modesty. Absence of a stressful environment is important for a positive outcome for both mother and baby, and the nurse should strive to alleviate stress and maintain a harmonious environment. Many cultures will foster relationships, and visits from extended family members may be important. The patient may not participate in all aspects of the health care system due to cultural issues. PTS: 1 DIF: Cognitive Level: Application REF: Page 791 OBJ: 8 TOP: Cultural considerations KEY: Nursing Process Step: Intervention

16. Early in the first trimester, a woman complains of morning sickness. What does the nurse suggest to aid with the discomfort? a. Eating something with a high-fat content b. Eating dry crackers before getting up c. Eating three well-balanced meals d. Getting rest and taking antiemetics

ANS: B A remedy for morning sickness is to eat a few dry crackers before getting up. PTS: 1 DIF: Cognitive Level: Application REF: Page 783 OBJ: 7 TOP: Morning sickness KEY: Nursing Process Step: Implementation

14. The first-time mother has been told by the nurse that the first stage of labor is the longest. What would be an appropriate nursing intervention for comfort during this time? a. Cool fluids to drink b. A backrub in the sacral area c. Assisting to lie in a supine position d. Decreasing illumination in the room

ANS: B Backache in the sacral area is a common complaint during the first stage of labor. The keyword is "comfort" in the question. Providing a backrub is providing comfort to the laboring patient.

22. During labor, the patient screams at her husband to get out of her sight. What would be the most appropriate action for the nurse? a. Ask the husband to leave the room b. Assure the husband that such behavior is normal c. Remind the patient that the husband wants to help d. Change the patient's position

ANS: B During labor the patient frequently becomes angry and outspoken. It is a normal occurrence, but the husband needs to be reassured that such behavior is normal.

19. A nurse is assessing the printout from the fetal monitor. What is the legal responsibility of the nurse? a. Correctly identifying abnormal FHR patterns and prescribing medication b. Correctly identifying abnormal FHR patterns and notifying the health care provider c. The nurse is not legally responsible for fetal monitoring d. Providing technical assessment to the monitor technicians

ANS: B Nurses are responsible for the timely notification of the primary caregiver in the event of an abnormal fetal heart rate (FHR) pattern. The nurse cannot write a medication order.

4. Why is the nurse concerned about a patient in her first trimester of pregnancy being exposed to German measles? a. The disease is capable of causing a spontaneous abortion. b. The disease is capable of causing birth defects. c. The disease is capable of causing high fever and convulsions. d. The disease is capable of interfering with placental implantation.

ANS: B Rubella is a known teratogen, which can cause birth defects. PTS: 1 DIF: Cognitive Level: Application REF: Page 764 OBJ: 1 TOP: Teratogen KEY: Nursing Process Step: Assessment

11. A woman asks the nurse about the safety of sexual intercourse during her pregnancy. Which response by the nurse is the most correct? a. "Sexual activity should be avoided after the first trimester." b. "Sexual activity should be ceased in the case of vaginal bleeding." c. "Sexual activity should be avoided in the second trimester." d. "Sexual activity should be limited to activity that does not include intercourse."

ANS: B Sexual intercourse can be enjoyed throughout pregnancy unless it is contraindicated by other conditions. In the case of vaginal bleeding, sexual activity should cease until the cause of the bleeding is determined by the doctor. PTS: 1 DIF: Cognitive Level: Analysis REF: Pages 787, 788 OBJ: 5 TOP: Sexual activity during pregnancy KEY: Nursing Process Step: Implementation

29. What should a nurse instruct the patient to do before assessing fundal height? a. Press her lower back against the examination table b. Empty her bladder c. Take a deep breath and hold it d. Bear down

ANS: B The bladder should be emptied before the measurement of the fundal height. PTS: 1 DIF: Cognitive Level: Application REF: Page 773 OBJ: 3 TOP: Fundal height KEY: Nursing Process Step: Assessment

18. The nurse is alarmed as she assesses a protruding umbilical cord from the vagina. What immediate action should the nurse take? a. Monitor intensity of contractions b. Place the patient in the knee-chest position c. Notify the charge nurse d. Ask the patient to perform a Valsalva maneuver

ANS: B The knee-chest position reduces the pressure on the prolapsed cord. The charge nurse will need to be notified, and the contractions will need to be monitored. However, the priority is reducing the pressure on the prolapsed cord.

7. What is the largest diameter of the fetal skull? a. Temporal b. Biparietal c. Lateral d. Frontal-occipital

ANS: B The largest transverse diameter of the fetal skull is the biparietal measurement. If this is too large, the skull may not be able to enter the mother's pelvis.

6. What protects the fetus from most bacterial infections? a. The yolk sac b. The placental barrier c. The cotyledons d. The chorionic villa

ANS: B The placental barrier protects the embryo/fetus from most bacteria, but not from viruses or drugs. The cotyledons are sections that make up the placenta. The chorionic villa are tiny vascular projections on the chorionic surface that help form the placenta. PTS: 1 DIF: Cognitive Level: Comprehension REF: Page 764 OBJ: 2 TOP: Placental barrier KEY: Nursing Process Step: Implementation

8. A woman who has just discovered she is pregnant states that the first day of her last menstrual period was July 10. What will be her expected date of birth (EDB)? a. April 10 b. April 17 c. May 10 d. October 17

ANS: B To determine the EDB (estimated date of birth), the woman should count from the first day of her last menstrual period. Count back 3 months and forward 7 days. PTS: 1 DIF: Cognitive Level: Application REF: Page 780 OBJ: 4 TOP: Estimated date of birth (EDB) KEY: Nursing Process Step: Assessment

33. A pregnant woman is discussing her desire to have her baby in a birthing center. Which factors could exclude the patient from delivering in a birthing center? (Select all that apply.) a. The patient is a primigravida. b. The patient will be having a planned cesarean delivery. c. The mother has preeclampsia. d. The baby is a boy. e. The mother has no support system.

ANS: B, C Birthing centers are ideal only for women who are considered low risk. Cesarean deliveries would not be done in a birthing center. The mother with preeclampsia would be considered high risk and would probably be excluded from delivering in the birthing center. The number of previous pregnancies, sex of the baby, and mother's support system would not be factors considered when determining risk for delivering in a birthing center.

MULTIPLE RESPONSE 32. Which assessment findings suggest probable fetal distress? (Select all that apply.) a. Fetal heart rate (FHR) of 120 b. Meconium-stained amniotic fluid c. Decreased FHR during contractions d. Strong contractions 10 seconds apart e. Slow return of FHR to baseline

ANS: B, E Meconium-stained amniotic fluid and the slow return of the FHR to the baseline are indicative of fetal distress. All other options are normal.

9. Which is a positive sign of pregnancy? a. Positive pregnancy test b. Positive Chadwick sign c. Ultrasonic tracing of the fetus d. Positive Goodell sign

ANS: C A positive sign of pregnancy is an ultrasonic tracing of the fetus. A positive pregnancy test, positive Chadwick sign, and positive Goodell sign are all probable signs of pregnancy. PTS: 1 DIF: Cognitive Level: Comprehension REF: Page 780 OBJ: 4 TOP: Positive signs of pregnancy KEY: Nursing Process Step: Assessment

25. The physician decides to send the mother for a test to determine the fetal lung maturity. What is the name of this fetal well-being test? a. Biophysical profile b. Alpha-fetoprotein c. Amniocentesis d. Ultrasound

ANS: C Amniocentesis helps determine the maturity of the fetal lungs. PTS: 1 DIF: Cognitive Level: Knowledge REF: Page 773, Table 25-2 OBJ: 3 TOP: Amniocentesis KEY: Nursing Process Step: N/A

13. During which gestational week can a primigravida expect to first feel fetal movement? a. 8 b. 10 c. 16 d. 20

ANS: C At about 16 to 18 weeks, the sensation of the first movement is felt. PTS: 1 DIF: Cognitive Level: Knowledge REF: Page 779 OBJ: 4 TOP: Quickening KEY: Nursing Process Step: Implementation

18. A woman entering the 22nd week of pregnancy complains that she has become unsightly because of chloasma. What should the nurse recommend to reduce the appearance of the chloasma? a. Use heavy makeup b. Take extra doses of vitamin A c. Avoid exposure to the sun d. Reduce caffeine intake

ANS: C At week 22, skin pigment changes called chloasma are found. Avoiding exposure to the sun will reduce the pigmentation. PTS: 1 DIF: Cognitive Level: Analysis REF: Page 784 OBJ: 7 TOP: Chloasma KEY: Nursing Process Step: Implementation

2. How do Braxton-Hicks contractions, which may begin in the first trimester and become increasingly stronger during the pregnancy, differ from labor contractions? a. Last several minutes b. Are always regular c. Do not dilate the cervix d. Are only mild

ANS: C Braxton-Hicks contractions do not dilate the cervix. Braxton-Hicks contractions remain irregular, can range from mild to moderate in severity, and increase in duration as the pregnancy progresses.

3. When trying to differentiate false labor from true labor, the nurse realizes which of the following statements regarding true labor is correct? a. Discomfort of the contraction is in the fundus. b. Contractions do not follow a pattern. c. Contractions get stronger with ambulation. d. Contractions may stop with ambulation.

ANS: C Contractions get stronger with ambulation in true labor. True labor is also marked by the onset of regular, rhythmic contractions.

29. For the first hour following delivery, how often should the nurse assess the mother? a. Every 5 minutes b. Every 10 minutes c. Every 15 minutes d. Every 30 minutes

ANS: C During the first hour, assessments are done every 15 minutes.

23. A primigravida patient is admitted to the labor and delivery unit. During initial assessment, the baby is found to be engaged. Which statement is true? a. The narrowest diameter of the presenting part has reached the pelvic outlet. b. The descending part is being initiated through the midpelvis. c. The widest diameter of the presenting part crosses the pelvic inlet. d. The narrowest diameter of the presenting part is at the ischial spines.

ANS: C Engagement occurs when the biparietal diameter, which is the widest part of the fetal head, crosses the pelvic inlet.

1. Where does implantation of the fertilized ovum usually occur? a. Lower uterine wall b. Side of the uterus c. Fundus of the uterus d. Body of the uterus

ANS: C Implantation usually occurs in the fundus of the uterus. PTS: 1 DIF: Cognitive Level: Knowledge REF: Page 763 OBJ: 1 TOP: Implantation KEY: Nursing Process Step: Implementation

5. What method is used to visualize soft tissue and to determine adequacy of the pelvis with no detrimental effects to the fetus? a. Pelvimetry b. Palpation c. Ultrasonography d. X-ray

ANS: C In more than 20 years of use, ultrasonography has had no detrimental effects on the fetus. Pelvimetry and x-ray uses radiation to visualize bony prominences. Pelvimetry is not used in the pregnant patient due to detrimental effects to the fetus. Palpation does not allow for visualization of soft tissue.

17. A patient arrives at the hospital having contractions. How should the nurse determine that the patient is in true labor? a. There is no dilation b. The contractions are in the fundus c. The cervix has softened and effaced d. The contractions are irregular

ANS: C One sign of true labor is when the cervix has softened and effaced. True labor contractions are regular and rhythmic.

26. Why is oxytocin administered in the third stage of labor? a. To stimulate lactation b. To relieve postpartum pain c. To stimulate uterine contractions d. To sedate the mother so she can rest

ANS: C Oxytocin makes the uterus contract and reduces postpartum hemorrhage.

28. An infant presents 5 minutes after delivery with a heart rate of 105, is crying, has some flexion in the arms, sneezes, and has a pink body and blue limbs. What Apgar score should be assigned to this infant? a. 5 b. 7 c. 8 d. 10

ANS: C The Apgar scoring is: fetal heart rate (FHR) over 100 = 2; crying = 2; flexed arms = 1; sneeze = 2; pink body, blue limbs = 1

16. The patient's membranes have just ruptured. What is the first priority of the nurse? a. Turn the patient on the left side b. Perform a Nitrazine test c. Check the fetal heart rate (FHR) d. Perform a vaginal examination

ANS: C The FHR should be assessed immediately after rupture of the membranes to determine the well-being of the baby.

20. A mother is in early labor and asks the nurse how long the labor will last. The nurse explains that the first stage of labor lasts from the beginning of regular contractions until when? a. The cervix is completely effaced b. The baby is in position c. The cervix is fully dilated d. The woman begins pushing

ANS: C The first stage of labor begins with regular contractions and ends with complete dilation of the cervix.

9. What is the ideal attitude for the fetal body during labor? a. Extension b. Lateral c. Flexion d. Transverse

ANS: C The ideal attitude for the fetal body is flexion.

15. A woman is admitted in active labor, and the nurse assesses the fetal heart rate (FHR) at 124 bpm. What action should the nurse take based on the assessment? a. Position patient on her left side b. Start oxygen per nasal cannula c. Reassure the mother the rate is normal d. Notify the physician at once

ANS: C The normal FHR is 120 to 160 bpm. No interventions are required.

19. During the final weeks of pregnancy, urinary frequency may return due to the enlarged uterus, compressing the bladder against the pelvic bones. What does the nurse suggest to aid in relieving the urinary frequency? a. Decrease fluid intake b. Use the knee-chest position c. Sleep on her side d. Avoid fluid intake in evening

ANS: C The patient should decrease pressure on the bladder at night by sleeping on her side. Fluids should not be decreased unless directed by a physician. PTS: 1 DIF: Cognitive Level: Application REF: Page 787 OBJ: 7 TOP: Frequency KEY: Nursing Process Step: Implementation

5. Which hormone is secreted by the placenta? a. Follicle-stimulating hormone (FSH) b. Alpha-fetoprotein (AFP) c. Human chorionic gonadotropin (HCG) d. Luteinizing hormone (LH)

ANS: C The placenta functions as an endocrine gland, secreting estrogen, progesterone, and HCG. PTS: 1 DIF: Cognitive Level: Comprehension REF: Page 764 OBJ: 2 TOP: Placenta function KEY: Nursing Process Step: Implementation

8. A nurse is teaching a group of primigravidas that during delivery, pressure on the fetal skull may produce changes in the shape of the skull. What is the reshaping of the skull called? a. Pressure response b. Overlapping c. Molding d. Spacing

ANS: C The reshaping of the skull bones in response to pressure is called molding.

25. A mother has entered the second stage of labor. When does the second stage of labor end? a. When the mother begins to push b. When the baby's head crowns c. With delivery of the baby d. With delivery of the placenta

ANS: C The second stage of labor begins with complete dilation and ends with the birth of the baby.

6. What area of the uterus provides the force during a contraction? a. Lower portion b. Middle portion c. Upper portion d. Cervical portion

ANS: C The upper portion of the uterus provides the force during contractions.

27. Which of the following discomforts of a pregnant woman should be reported to the physician at the first occurrence? a. Leg cramps b. Pelvic discomfort c. Vaginal bleeding d. Urinary frequency

ANS: C Vaginal bleeding at any time during pregnancy should be reported to the physician. Leg cramps, pelvic discomfort, and urinary frequency are common discomforts of pregnancy and not a cause for immediate concern. PTS: 1 DIF: Cognitive Level: Application REF: Page 788 OBJ: 5 TOP: Danger indicators KEY: Nursing Process Step: Implementation

20. A pregnant teenager presents with the following complaints. Which complaint could be an indicator of a serious complication? a. Painful hemorrhoids b. Linea nigra c. Visual disturbances d. Low back pain

ANS: C Visual disturbances may be an indicator of increased blood pressure and retained fluids. These are indicators of eclampsia. Hemorrhoids, linea nigra, and back pain are common discomforts of pregnancy. PTS: 1 DIF: Cognitive Level: Analysis REF: Page 783, Box 25-10 OBJ: 5 TOP: Danger signs KEY: Nursing Process Step: Assessment

2. A patient has been diagnosed with a tubal pregnancy. What is the typical outcome of a tubal pregnancy? a. The patient will carry the pregnancy to term and have a cesarean delivery. b. The patient will have to remain in bed for the remainder of the pregnancy. c. The patient will spontaneously abort this ectopic pregnancy. d. The patient will require surgery to remove the zygote.

ANS: D Any pregnancy where implantation occurs outside the uterine cavity is called ectopic. Tubal pregnancies usually must be resolved by surgical removal of the zygote. PTS: 1 DIF: Cognitive Level: Analysis REF: Page 763 OBJ: 1 TOP: Pregnancy KEY: Nursing Process Step: Assessment

21. The nurse is admitting a patient to the labor and delivery unit. While performing the initial assessment, which assessment is the priority? a. The number of previous pregnancies b. When the baby is due c. When the patient last ate d. The timing of contractions

ANS: D Assessment begins with timing the contractions on admission to form a database.

22. The newly diagnosed primigravida who is 6 weeks pregnant states, "I don't feel like I have a real baby inside me." To reassure the mother, the nurse provides reassurance that which of the following is functioning in the 6-week-old embryo? a. Brain b. Lungs c. Hands d. Heart

ANS: D At 6 weeks, the fetus has a pumping heart. PTS: 1 DIF: Cognitive Level: Comprehension REF: Page 765, Table 25-1 OBJ: 1 TOP: Fetal development KEY: Nursing Process Step: Implementation

24. When can the sex of the fetus be confirmed? a. Conception b. 2 weeks c. 6 weeks d. 9 weeks

ANS: D At 9 weeks the genitalia are well defined. PTS: 1 DIF: Cognitive Level: Knowledge REF: Page 766, Table 25-1 OBJ: 1 TOP: Fetal sex determination KEY: Nursing Process Step: Implementation

14. At what week of fetal development can the nurse expect to first hear fetal heart tones with an amplified stethoscope? a. 10 b. 12 c. 14 d. 16

ANS: D During week 16, the fetal heart can be heard with an amplified stethoscope. PTS: 1 DIF: Cognitive Level: Knowledge REF: Page 780, Box 25-5 OBJ: 4 TOP: Fetal age KEY: Nursing Process Step: Implementation

10. What is the cause of frequent urination in early pregnancy? a. Increased fluid intake b. The fetus's kidneys functioning c. Retention of fluid d. Increased circulating volume

ANS: D Early in pregnancy, the increase in circulating volume and the enlarging uterus placing pressure on the bladder cause urinary frequency. PTS: 1 DIF: Cognitive Level: Application REF: Page 786 OBJ: 7 TOP: Frequency of urination KEY: Nursing Process Step: Implementation

12. Which type of monitor will assesses the intensity of contractions? a. External monitor b. Fetal monitor c. Maternal monitor d. Internal monitor

ANS: D Internal monitoring is used to monitor the intensity of contractions, the frequency and duration of contractions, and the resting tone of uterine contractions. An external monitor is used to monitor the fetal heart rate and uterine activity.

17. What does the increase in circulating blood volume during pregnancy cause in the mother? a. Shortness of breath b. Frontal headaches c. Decreased white blood cell count d. Decreased hemoglobin

ANS: D Maternal circulating volume increases 30% to 40%, causing a virtual decrease in hemoglobin. PTS: 1 DIF: Cognitive Level: Analysis REF: Page 786, Table 25-5 OBJ: 2 TOP: Decreased Hgb KEY: Nursing Process Step: Implementation

23. Smoking by the mother can have what effect in the fetus? a. Hearing deficits b. Neuromuscular deformities c. Cerebral palsy d. Low birth weight

ANS: D Smoking has been proven to cause slow intrauterine growth and low birth weight. PTS: 1 DIF: Cognitive Level: Application REF: Page 782 OBJ: 5 TOP: Smoking KEY: Nursing Process Step: Implementation

12. A woman tells the nurse that this is her third pregnancy. She has had twin girls at full term and one miscarriage. How does the nurse record the information? a. G2, T2, L3 b. G4, T3, A1, L1 c. G3, T3, A2, L1 d. G3, T1, A1, L2

ANS: D Standard obstetrical terminology is: G = gravida, T = term birth, P = preterm birth, A = abortion, L = living children. PTS: 1 DIF: Cognitive Level: Comprehension REF: Pages 780-781, Box 25-6 OBJ: 3 TOP: Terminology KEY: Nursing Process Step: Assessment

28. What do the arteries in the umbilical cord carry? a. Nutrients to the fetus from the placenta b. Oxygenated blood to perfuse the placenta c. Antibodies from the fetus to the mother d. Deoxygenated blood back to the placenta

ANS: D The arteries of the umbilical cord are unique in that they carry deoxygenated blood back to the placenta. PTS: 1 DIF: Cognitive Level: Comprehension REF: Page 772 OBJ: 2 TOP: Umbilical arteries KEY: Nursing Process Step: Implementation

3. How long does the embryonic stage of pregnancy typically last? a. 3 weeks b. 4 weeks c. 6 weeks d. 8 weeks

ANS: D The embryonic stage encompasses the first 8 weeks. PTS: 1 DIF: Cognitive Level: Knowledge REF: Page 764 OBJ: 1 TOP: Pregnancy KEY: Nursing Process Step: Implementation

7. What period of the maternity cycle does the intrapartal period cover? a. Beginning of pregnancy to midterm b. Conception to third trimester c. Onset of labor to delivery of the baby d. Onset of labor to delivery of the placenta

ANS: D The intrapartal period of the maternity cycle covers the onset of labor to delivery of the placenta. The antepartal period begins at conception and continues until the onset of labor. The postpartal period begins after the delivery of the placenta and continues for approximately 6 weeks, until the reproductive organs return to their prepregnancy state. PTS: 1 DIF: Cognitive Level: Knowledge REF: Page 778 OBJ: 3 TOP: Intrapartal period KEY: Nursing Process Step: Implementation

15. The nurse assures an anxious primigravida that during fetal development from week 34 and beyond, maternal antibodies are transferred to the baby. How long will these antibodies provide the baby with immunity? a. 1 month b. 3 months c. 4 months d. 6 months

ANS: D The maternal antibodies that are transferred to the baby provide immunity for 6 months. PTS: 1 DIF: Cognitive Level: Application REF: Page 771, Table 25-1 OBJ: 2 TOP: Pregnancy KEY: Nursing Process Step: Implementation

27. After the delivery of a newborn, what is the priority action of the nurse? a. Place the newborn on the right side b. Cover the cord stump c. Dry the infant immediately d. Suction nose and mouth

ANS: D To prevent aspiration of amniotic fluid, the baby should be suctioned, then quickly dried to prevent hypothermia.

31. In the illustration below, which item depicts the LOT position? a. 1 b. 2 c. 3 d. 4 e. 5 f. 6

ANS: E The LOT position is left occiput transverse.

The newlywed couple most appropriately referred to genetic counseling would be the: a. Jewish couple who feel they must have a male child. b. Caucasian couple who both have family histories of schizophrenia. c. mixed cultural couple of a Japanese wife and American husband. d. African Americans, one of whom has sickle cell trait.

African Americans, one of whom has sickle cell trait.

A nurse is caring for a client diagnosed with abruptio placentae. During labor, the priority nursing action would be to monitor:

All vital signs, especially heart rate and blood pressure

A nurse is teaching a prenatal class on the anatomy and physiology of the female reproductive system. When a participant in the class asks where the follicle-stimulating hormone is produced, the nurse responds that it is produced in the:

Anterior pituitary gland

During the intrapartum period, the nurse assists the health care team to ensure appropriate intravenous (IV) fluid intake and oxygen consumption for the laboring client with sickle cell disease. This action will primarily:

Assist in preventing dehydration and hypoxemia.

A client in active labor with intact membranes is complaining of back discomfort. An analgesic was administered 1 hour ago but has not relieved the discomfort. The nurse should avoid doing which of the following at this time to assist in relieving the back discomfort?

Assist the client to ambulate in the room.

A 31-week preterm labor client dilated to 4 centimeters has been started on magnesium sulfate. Her contractions have stopped. If the client's labor can be inhibited for the next 48 hours, what medication does the nurse anticipate will be prescribed?

Betamethasone

A pregnant client asks the nurse in the clinic when she will be able to start feeling the fetus move. The nurse responds by telling the mother that fetal movements will be noted:

Between 16 and 20 weeks' gestation

The nurse is collecting data from a client who has been diagnosed with placenta previa. Choose the findings that the nurse would expect to note. Select all that apply.

Bright red vaginal bleeding Soft, relaxed, nontender uterus

Before attempting to deliver the placenta after a precipitate delivery, the nurse waits for which sign as an indication of placental separation?

Change in uterine shape

A nurse is caring for a client with a diagnosis of dystocia. The nurse specifically collects data regarding which of the following?

Characteristics of contractions

A client is brought to the labor unit, and, as the nurse is attaching the fetal heart monitor, the client's membranes rupture spontaneously. The nurse immediately:

Checks the fetal heart rate

A nurse assists the nurse-midwife to examine the client. The midwife documents the following data: cervix 80% effaced and 3 cm dilated, vertex presentation minus (−) 2 station, membranes ruptured. The nurse anticipates that the midwife will prescribe which of the following activity for the client?

Complete bedrest

A nurse reviews the results of an ultrasound performed on a woman admitted to the maternity unit. The results indicate that the placenta is covering the entire internal cervical os. The nurse understands that the client is experiencing:

Complete placenta previa

A client age 23 develops melasma during pregnancy. The nurse notes that the client has started wearing very heavy makeup. The client tells the nurse that she is fearful that her mate will reject her, and that she has decreased her social engagements drastically because of this change. The nurse determines that the client is experiencing which problem?

Concern about her appearance

A nurse is caring for a client in labor. The fetal heart rate is 156 beats per minute and regular. The client's contractions are occurring every 4 minutes with a duration of 42 seconds and moderate intensity. The nurse should do which of the following at this time?

Continue monitoring the client because the data reflect acceptable progress.

The client is in the second stage of labor. As the baby begins to crown, the health care provider administers a pudendal nerve block in preparation for an episiotomy. The nurse should:

Continue to assess vital signs and fetal heart rate the same as before the nerve block.

The client is having moderate contractions that are occurring every 5 minutes and lasting 60 seconds. The fetal heart rate (FHR) is 150 beats per minute and regular. Based on these findings, what is the appropriate nursing action?

Continue to monitor the client.

A nurse is assigned to care for a nulliparous client who is having a precipitate delivery. The nurse reports which maternal focused observations?

Decreased periods of uterine relaxation between contractions

A nurse is monitoring a client in labor whose membranes rupture spontaneously. The initial nursing action is to:

Determine the fetal heart rate.

A primigravida's membranes rupture spontaneously. The nurse's first action is to:

Determine the fetal heart rate.

A nurse in the labor room is caring for a client in the first stage of labor. On assessing the fetal patterns, the nurse notes an early deceleration of the fetal heart rate (FHR) on the monitor strip. Which is the appropriate nursing action?

Document the findings and continue to monitor the fetal patterns.

A nurse is providing dietary instructions to a pregnant client with a history of lactose intolerance. The nurse would instruct the client to consume which best food item to ensure an adequate source of calcium in the diet?

Dried fruits

The advantages of using spinal anesthesia for delivery of a fetus include which of the following? Select all that apply.

Ease of administration Absence of fetal hypoxia Immediate onset of anesthesia

A client is scheduled to have an elective cesarean delivery. The nurse preparing the client for the procedure plans to allay the client's feelings of anxiety by:

Encouraging the client to discuss her concerns and desires regarding anesthesia options

A nurse assisting to monitor a client in labor is told that the client's cervix is 3 cm dilated with contractions occurring every 2 to 3 minutes. When monitoring the client's psychological status, the nurse anticipates the client to reflect an attitude of:

Excitement

A nurse is providing emergency measures to a pregnant client with a prolapsed cord. The mother becomes anxious and frightened and says to the nurse, "Why are all of these people in here? Is my baby going to be all right?" Which of the following most appropriately describes the mother's problem at this time?

Fear about what is happening

A nurse observes that a client in the transition stage of labor is crying out in pain with pushing efforts. The nurse recognizes this behavior as:

Fear of losing control

A client who is 6 months pregnant is attending her first prenatal visit. On the first prenatal visit, the nurse notes that the client is gravida IV, para 0, aborta III. The client is 5 feet, 6 inches tall, weighs 130 pounds, and is 25 years old. She states, "I get really tired after working all day and can't keep up with my housework." Which factor in the above data would lead the nurse to suspect gestational diabetes?

Fetal demise

A licensed practical nurse (LPN) is assisting in gathering data on a client who is scheduled for a cesarean delivery. Which of the following findings would indicate a need to contact the registered nurse (RN)?

Fetal heart rate of 180 beats per minute

A nurse is monitoring a client who is receiving oxytocin (Pitocin) to augment labor. The nurse determines that the dosage should be decreased and notifies the registered nurse if which of the following is noted?

Fetal tachycardia

A nurse is reading the health care provider's (HCP) documentation regarding a pregnant client and notes that the HCP has documented that the client has a platypelloid pelvic shape. The nurse understands that this pelvic shape is:

Flat and nonfavorable for a vaginal birth

A nurse is monitoring a preterm labor client who is receiving magnesium sulfate intravenously. The nurse monitors for which adverse effect(s) of this medication? Select all that apply.

Flushing Depressed respirations Extreme muscle weakness

Of the following, which would be the appropriate method to use to deliver the placenta after a precipitate delivery?

Gently guide the placenta out after a spontaneous separation.

A pregnant client tells the nurse that she felt wetness on her peri-pad and that she found some clear fluid. The nurse immediately inspects the perineum and notes the presence of both a clear liquid and a portion of the umbilical cord. The nurse's initial action is to:

Gently hold the presenting part upward.

A pregnant client is positive for the human immunodeficiency virus (HIV). Based on this information, the nurse determines that:

HIV antibodies are detected on the enzyme-linked immunosorbent assay (ELISA) test.

A nurse is asked to assist the primary health care provider in performing Leopold's maneuvers on a client. Which nursing intervention should be implemented before this procedure is performed?

Have the client empty her bladder.

A client calls the health care provider's office to schedule an appointment because a home pregnancy test was performed and the results were positive. The nurse determines that the home pregnancy test identified the presence of which of the following in the urine?

Human chorionic gonadotropin (hCG)

For the previous 4 hours, a client in labor has been experiencing contractions every 2 minutes, lasting 60 to 70 seconds, and strong to palpation. She is 2 cm dilated and complaining of severe pain. The nurse understands that the client is experiencing which type of dystocia?

Hypertonic

A prenatal client with severe abdominal pain is admitted to the labor and birthing department. Which data indicate to the nurse the presence of concealed bleeding?

Increase in fundal height

A client arrives at the birthing center in active labor. Her membranes are still intact and the nurse-midwife performs an amniotomy. The nurse explains to the client that after this procedure, she will likely have:

Increased efficiency of contractions

A nurse is collecting data on a pregnant woman who is human immunodeficiency virus (HIV) positive during the 32nd gestational week. The nurse reviews the data and determines that which finding requires further follow up?

Increased shortness of breath and bilateral rales

A nurse assisting in the care of a woman in labor should focus primarily on which of the following at the time of delivery?

Infant

Leopold's maneuvers will be performed on a pregnant client. The client asks the nurse about the procedure. The nurse responds, knowing that this procedure:

Is a systematic method for palpating the fetus through the maternal abdominal wall

The nurse prepares to administer erythromycin ophthalmic ointment to a newborn infant immediately after delivery. The nurse understands that this ointment:

Is effective in protecting the newborn from Neisseria gonorrhoeae and chlamydia

A nurse prepares to explain the purpose of effleurage to a client in early labor. The nurse tells the client that effleurage:

Is light stroking of the abdomen to facilitate relaxation during labor

Which documentation concerning the characteristics of amniotic fluid supports the determination that the fluid is normal?

It is pale, straw-colored with flecks of vernix.

A pregnant client in the prenatal clinic states that her last menstrual period (LMP) began April 5 and ended April 12. According to Nägele's rule, what would be the estimated date of delivery (EDD)?

January 12

A nurse is caring for a woman in labor who is experiencing a precipitate delivery. Until help arrives, the nurse places the client into which optimal position?

Lateral Sims'

A client in preterm labor is placed on bedrest. The nurse assists the client to which of the following advantageous positions?

Left lateral

The nurse is caring for a client who is in labor. The nurse rechecks the client's blood pressure and notes that it has dropped. To decrease the incidence of supine hypotension, the nurse should encourage the client to remain in which position?

Left lateral

A nurse has assisted in developing a plan of care for a client experiencing dystocia and includes several nursing interventions in the plan of care. The nurse prioritizes the plan and selects which nursing intervention as the highest priority?

Monitoring fetal status

A pregnant client at 36 weeks' gestation experiences painless bleeding and is admitted to the labor room. Which action should the nurse initially include in the plan of care?

Maintain complete bedrest, monitor IV fluid intake, and monitor the fetal heart rate.

A nurse is assisting in preparing to care for a client undergoing an induction of labor with an infusion of oxytocin (Pitocin). The nurse includes which of the following in the plan of care?

Maintain continuous electronic fetal monitoring.

The maternity nurse prepares the client for which of the following techniques commonly used to relieve shoulder dystocia?

McRoberts' maneuver

The client at 38 weeks' gestation is admitted to the birthing center in early labor. The client is carrying twins, and one of the fetuses is in a breech presentation. The nurse assists with planning care for the client and identifies which of the following as the lowest priority for the care of this client?

Measuring the fundal height

A nurse is assigned to care for a client experiencing dystocia. In planning care, the nurse would consider the highest priority to be frequent:

Monitoring for changes in the physical and emotional condition of the mother and fetus

The nurse is reviewing the health record of a pregnant client at 16 weeks' gestation. The nurse should expect to note documentation that the fundus of the uterus is located at which of the following areas?

Midway between the symphysis pubis and the umbilicus

A nurse encourages the childbearing woman diagnosed with human immunodeficiency virus (HIV) HIV to avoid alcohol and cigarettes during pregnancy and to obtain adequate rest. Which outcome is specific to this client?

Minimize the potential for developing infections.

A nurse is caring for a client in preterm labor when her membranes rupture. The initial nursing action is to:

Monitor the fetal heart rate.

A nurse tells a client she is now beginning the second stage of labor. The nurse realizes the client understands the occurrences of this stage when the client says:

My cervix is completely dilated."

A woman in active labor has contractions every 2 to 3 minutes that last for 45 seconds. The fetal heart rate between contractions is 100 beats per minute. On the basis of these findings, the priority nursing intervention is to:

Notify the registered nurse (RN) immediately.

A nurse is monitoring a client in active labor and notes that the client is having contractions every 3 minutes that last 45 seconds. The nurse notes that the fetal heart rate between contractions is 100 beats per minute. Which of the following nursing actions is appropriate?

Notify the registered nurse (RN).

A nurse notes that a client in labor has foul-smelling amniotic fluid, a maternal temperature of 101° F, and a urine output of 150 mL during the past 2 hours. The nurse should do which of the following at this time?

Notify the registered nurse of a possible maternal infection.

A nurse is evaluating the effectiveness of meperidine hydrochloride (Demerol) for pain management for a client in labor. The client describes her pain level as "9" during contractions. The nurse determines that the medication was effective if the client exhibited which reasonable goal for pain relief?

Pain level is "4" while a progressive labor pattern continues.

A prenatal client with vaginal bleeding is admitted to the labor unit. Which of the following signs or symptoms indicates placenta previa?

Painless vaginal bleeding

A nurse is providing instructions to a pregnant woman regarding measures that will strengthen the perineal floor muscles. The nurse instructs the client to:

Perform Kegel exercises in 10 repetitions, three times per day.

A nursing instructor asks a nursing student to describe the procedure for relieving an airway obstruction on an unconscious pregnant woman at 8 months' gestation. The student describes the procedure correctly if the student states to:

Place a rolled blanket under the right abdominal flank and hip area.

A nurse is caring for a woman in the delivery room. The health care provider prescribes an oxytocic medication for the woman to stimulate uterine contractions and prevent hemorrhage. The nurse understands that this medication will be administered after delivery of the:

Placenta

A nurse should prepare to give a prescribed oxytocic medication after delivery of the:

Placenta

After the client vaginally delivers a viable newborn, the nurse sees the umbilical cord lengthen and observes a spurt of blood from the vagina. The nurse recognizes these findings as signs of:

Placental separation

A nurse is assisting to care for a pregnant client in labor who will be delivering twins. The nurse prepares to monitor the fetal heart rates by:

Placing external fetal monitors so that each fetal heart rate is monitored separately

A nurse caring for a client diagnosed with placental abruption would plan to:

Prepare the client for a cesarean birth.

A client was admitted to the maternity unit 12 hours ago and has been experiencing strong contractions every 3 minutes, and the fetus is currently at station 0. The fetal heart rate on admission was 140 beats per minute and regular. The fetal heart rate is decreasing and a persistent nonreassuring fetal heart rate pattern is present. What is the appropriate nursing action?

Prepare the client for a cesarean delivery.

At 5:00 AM a client is admitted to the maternity unit after experiencing 3 hours of labor at home. The assessment determines that the fetal heart rate (FHR) is 140 beats per minute with the fetus at station 0 and strong contractions occurring every 3 minutes. It is now 7:00 AM with little progress, and the FHR is decreasing. It is most appropriate for the nurse to anticipate the need to:

Prepare the client for a cesarean delivery.

The nurse institutes measures for the client with placental abruption to minimize alterations in fetal tissue perfusion. The nurse determines that fetal tissue perfusion is adequate if which of the following is noted?

Presence of accelerations

A nurse is caring for a client with sickle cell disease who is in labor. The nurse ensures that the client receives appropriate intravenous (IV) fluid intake and oxygen consumption to primarily:

Prevent dehydration and hypoxemia.

A nurse is assigned to assist in caring for a client in labor. The nurse would determine that which of the following would least likely indicate dystocia?

Progressive changes in the cervix

A maternity nurse is describing the ovarian cycle to a group of nursing students and asks a nursing student to identify the phases of the cycle. Which phase, if stated by the nursing student, indicates a need to further research this area?

Proliferative phase

A nurse assisting in the labor room is preparing to care for a client with hypertonic dysfunction. The nurse is told that the client is experiencing uncoordinated contractions that are erratic in their frequency, duration, and intensity. The priority nursing intervention in caring for the client is to:

Provide pain relief measures.

A nurse is assisting in planning care to meet the emotional needs of a pregnant woman. Which of the following nursing interventions would be least likely to assist in meeting her emotional needs?

Providing the mother with pamphlets and booklets to read about the pregnancy

Immediately following the delivery of a newborn, the nurse prepares to assist in the delivery of the placenta. What is the appropriate action to deliver the placenta?

Pull gently on the cord as the mother bears down.

A nurse is preparing to collect data on a client with a possible diagnosis of ectopic pregnancy. Which of the following would the nurse check first?

Pulse

If a precipitate delivery is imminent, which of the following would be the appropriate nursing action?

Put on sterile gloves, and gently guide the baby's head and shoulders out.

A nurse is caring for a client following a precipitate delivery. In addition to fundal massage, the nurse understands that which nursing action will promote the birth of the placenta?

Putting the baby to the mother's breast and letting the baby suck

A nurse assists a pregnant client with cardiac disease to identify resources to help her care for her 18-month-old child during the last trimester of pregnancy. The nurse encourages the pregnant client to use these resources primarily to:

Reduce excessive maternal stress and fatigue.

A nurse is preparing a client for a cesarean delivery. A urinary catheter is to be inserted into the client's bladder, and the client asks the nurse why this is necessary. The nurse appropriately replies by telling the client that its primary purpose is to:

Reduce the risk of injuring the bladder during the surgery.

A client is admitted for an emergency cesarean section delivery. Contractions are occurring every 15 minutes. The client has a temperature of 100° F and ate 2 hours ago. Which intervention has priority?

Report the time of last food intake to the health care provider

A client who is a primigravida is receiving magnesium sulfate for gestational hypertension. The nurse is asked to monitor the client every 30 minutes. Which of the following information would be of concern to the nurse?

Respirations of 10 breaths per minute

A pregnant client is receiving magnesium sulfate for the management of preeclampsia. A nurse determines that the client is experiencing toxicity from the medication if which of the following is noted on data collection?

Respirations of 10 breaths per minute

A client in labor has been pushing effectively for 1 hour and the presenting part is at a +2 station. The nurse determines that the client's primary physiological need at this time is:

Rest between contractions

A nurse is collecting data from a client on her first prenatal visit. Which factor indicates that the client is at risk for developing gestational diabetes during this pregnancy?

She has a history of chronic hypertension.

A nurse is assisting in caring for a client in labor. The nurse recognizes that the risks for uterine rupture during labor and delivery include:

Shoulder dystocia

A nurse is monitoring the status of a client in active labor. The nurse interprets that which finding is consistent with dystocia? Select all that apply.

Signs of fetal distress High level of maternal anxiety Failure of the fetus to descend

A nurse assists in developing a plan of care for a multigravida client who has a history of cesarean birth. It is determined that the client is at high risk of uterine rupture. The nurse plans to monitor the client closely for:

Signs of shock

A nurse caring for a client who is receiving oxytocin (Pitocin) for the induction of labor notes a nonreassuring fetal heart rate (FHR) pattern on the fetal monitor. On the basis of this finding, the nurse would first:

Stop the oxytocin infusion.

A client in labor is transported to the delivery room and is prepared for a cesarean delivery. The client is positioned on the delivery room table and the nurse places the client in the:

Supine position with a wedge under the right hip

The client who is being prepared for a cesarean delivery is brought to the delivery room. To maintain the optimal perfusion of oxygenated blood to the fetus, the nurse places the client in the:

Supine position with a wedge under the right hip

A nurse is assisting in caring for a client with abruptio placentae and is monitoring the client for disseminated intravascular coagulopathy (DIC). Which of the following findings is least likely associated with DIC?

Swelling of the calf of one leg

A pregnant client is seen in the health care clinic for a regular prenatal visit. The client tells the nurse that she is experiencing irregular contractions. The nurse determines that the client is experiencing Braxton Hicks contractions. Based on this finding, which nursing action is appropriate?

Tell the client that these are common and they may occur throughout the pregnancy.

The nurse is doing a 48-hour postpartum check on a client with mild gestational hypertension (GH). Which of the following data indicate that the GH is not resolving?

The client complains of a headache and blurred vision.

During a routine prenatal visit the client states, "I have not been able to get my wedding ring off for the past 2 days. I guess the heat is making my fingers swell." The nurse needs to further check:

The client for blood pressure changes and protein in the urine

A pregnant client is newly diagnosed as having gestational diabetes. She cries during the interview and keeps repeating, "What have I done to cause this? If I could only live my life over." Which client problem should initially direct nursing care at this time?

The client is blaming herself.

A client becomes increasingly more anxious and hyperventilates during the transition phase of labor. The nurse recognizes that the client needs:

To regain her breathing pattern

A nurse is assisting in teaching a series of classes on maintaining a healthy pregnancy. The goal for the class is "The pregnant woman will verbalize measures that may prevent physical traumatic conditions distressing to the fetus." Based on this goal, which of the following would be a part of the teaching plan for this class?

Travel precautions and use of shoulder seat belts

A nurse collecting data on a client during the second stage of labor notes a slowing of the fetal heart rate (FHR) with a loss of variability and determines that these are indicators of possible complications. Which priority interventions should the nurse perform?

Turn client to her side and administer oxygen by mask at 8 to 10 L/min.

In providing initial care to the newborn following delivery, the priority action of the nurse is to:

Turn the infant's head to the side.

When examining the umbilical cord immediately after birth, the nurse expects to observe:

Two arteries

The nurse is describing the process of fetal circulation to a client during a prenatal visit. The nurse tells the client that fetal circulation consists of:

Two umbilical arteries and one umbilical vein

A nurse collects data from a pregnant client diagnosed with iron deficiency anemia during her third trimester for additional risk factors associated with the anemia. Which finding would support potential further maternal compromise?

Vaginal spotting twice since the last prenatal visit

A nurse is collecting data from a pregnant client with a history of cardiac disease and is checking the client for venous congestion. The nurse inspects which body area, knowing that venous congestion is commonly noted in this area?

Vulva

A nurse is collecting data from a pregnant client with a history of cardiac disease. The nurse is checking for venous congestion. The nurse inspects which of the following areas, knowing that venous congestion is most commonly noted here?

Vulva

During the first trimester of pregnancy, a client complains of frequent nausea followed by vomiting. On data collection, which finding would indicate a serious nutritional disorder of pregnancy?

Weight compared to last visit is a loss of 2.3 pounds.

A pregnant client in the third trimester of pregnancy with a diagnosis of mild preeclampsia is being monitored at home for progression of the disease process. The home care nurse teaches the client about the signs that need to be reported to the health care provider (HCP) and tells the client to call the HCP if:

Weight increases by more than 1 pound in a week.

A nurse is preparing a client for an emergency cesarean delivery. Which of the following information regarding the client has priority?

When was the last time the client ate or drank?

Warnings that are appropriate to an exercising pregnant woman in her second trimester of pregnancy would include: a. limiting water while exercising to prevent hypertension. b. avoiding exercises that require the supine position. c. performing exaggerated range-of-motion exercises. d. exercising strenuously to supply extra heat and blood to the placenta.

avoiding exercises that require the supine position.

A 30-week gestational prenatal client with complaints of painless vaginal bleeding presents at the labor and birthing department of the hospital. The nurse prepares the client for which expected diagnostic procedure?

contraction stress test

The pH of the female reproductive tract and estrogen level have an effect on fertilization in that they: a. soften the ovum to enhance fertilization. b. can trap defective sperm. c. determine the sex of the fetus. d. affect the speed of the ova and sperm entering the fallopian tube.

can trap defective sperm.

A pregnant client has been diagnosed with placental abruption. The nurse caring for the client prepares the client for:

cesarean birth

A nurse in the delivery room is assisting with the delivery of a newborn. Which observation would indicate that the placenta has separated from the uterine wall and is ready for delivery?

changes in the shape of the uterus

Good prenatal nutrition is stressed to an expectant mother because poor nutrition could increase the baby's risk of _____________ in later life. a. diabetes b. kidney failure c. respiratory disorders d. coronary artery disease

coronary artery disease

The nurse is assigned to assist with caring for a client who is being admitted to the birthing center in early labor. On admission, the nurse would initially:

determine the maternal and fetal vital signs.

The home health-care worker visiting a pregnant woman in her second trimester observes the patient doing chores and performing errands. Because of a threat to the fetus, the woman should refrain from: a. walking a mile to the grocery store. b. sweeping the floor with a broom. c. using bleach when doing the wash. d. emptying and cleaning the cat box.

emptying and cleaning the cat box.

After delivery, the new father is staring intently at his newborn and lightly running his fingertips over the baby's face. The father is beginning: a. parenting. b. engrossment. c. development. d. identification.

engrossment.

The ELSI program has been developed for the purpose of: a. promoting fund raising for the study of gene therapy. b. publishing up-to-date information on the progress of gene therapy. c. evaluating the ethical, legal, and social implications of gene therapy. d. educating the general public about the benefits of gene therapy.

evaluating the ethical, legal, and social implications of gene therapy.

Development of neural tube defects seems to be more common in the fetuses of pregnant women whose diet is low in: a. iron. b. vitamin D. c. folic acid. d. essential amino acids.

folic acid.

The Human Genome Project is credited with the development of significant research on the subject of: a. analysis of chemicals that make up RNA. b. prevention of birth defects. c. ethical use of stem cells. d. gene mapping.

gene mapping.

A complete set of DNA contained in all human cells is referred to as a(n) __________.

genome

A nurse is assigned to assist in preparing a woman who is gravida VI for delivery. In planning care for this client, the nurse places which of the following at the client's bedside?

intravenous (IV) supplies

The prenatal intervention that would be most helpful to a prospective father about his new role would be to: a. explain the physiological and psychological changes he can expect in the mother. b. demonstrate diapering and bathing techniques. c. let him verbalize his feelings about the pregnancy and fatherhood. d. encourage him to think about his role in the pregnancy.

let him verbalize his feelings about the pregnancy and fatherhood.

According to parental role development theory, the reality of the pregnancy occurs for the couple when: a. the mother is aware that she has missed her period. b. the pregnancy is confirmed in the first trimester. c. movement is felt in the second trimester. d. actual plans for the birth are made in the third trimester.

movement is felt in the second trimester.

The single most important influence on appropriate growth and disease prevention at all stages of development is: a. nutrition. b. heredity. c. culture. d. environment.

nutrition.

Methods of holding a neonate that are comforting and consoling to the baby and reinforce bonding include: (Select all that apply.) a. holding baby upright on parent's knee. b. placing the nude baby on the naked chest of the parent. c. swaddling the baby. d. cuddling the baby. e. the football hold supporting baby with one arm.

placing the nude baby on the naked chest of the parent. swaddling the baby. cuddling the baby.

The home health-care worker can help a father in his transition to parenthood by: a. pointing out that the infant recognizes his voice. b. asking him what child care arrangements have been made. c. teaching him to tape the baby's diaper a different way. d. reminding him that he must be ready to help with household chores.

pointing out that the infant recognizes his voice.

During a telephone follow-up conversation 5 days postpartum, the woman tells the health-care worker, "I don't know what's wrong with me. I love my baby, but I seem to be crying all the time!" The woman is most likely experiencing: a. taking-in. b. letting go. c. postpartum blues. d. attachment.

postpartum blues.

After the delivery of the newborn, the major focus for its well-being is to: a. initiate hydration. b. provide nutrition. c. prevent heat loss. d. stimulate bonding.

prevent heat loss.

The most effective time for interaction between parents and the new baby is when the baby is in the state of: a. active alert. b. crying. c. drowsiness. d. quiet alert.

quiet alert.

The health-care worker observes several interactions between a postpartum woman and her newborn. The behavior that would be assessed as maladaptive is: a. seldom making eye contact with the baby. b. reading a magazine while the baby sleeps. c. cuddling the baby close to her chest. d. telling visitors how well the baby is feeding.

seldom making eye contact with the baby.

A 15-year-old mother seems to ignore her newborn. A strategy that the health-care worker can use to facilitate mother-infant attachment in this mother is to: a. tell the mother that she must pay attention to her baby. b. show the mother how the baby initiates interaction in face-to-face encounters. c. demonstrate for the mother different positions for holding her baby while feeding. d. arrange for the mother to watch a video on parent-infant interaction.

show the mother how the baby initiates interaction in face-to-face encounters.

"Taking hold" is occurring when the new mother: a. requests help with ambulation and perineal care. b. shows interest in learning more about infant care. c. sleeps most of the time when the baby is not present. d. is very excited and talkative about the birth experience.

shows interest in learning more about infant care.

Characteristics of dizygotic twins include their being: a. of the same sex. b. formed from the same single ovum and sperm. c. always carried to full term. d. similar to each other as any sibling.

similar to each other as any sibling.

The difference between a nurse midwife and a doula is that a doula is a: a. registered nurse with advanced training in labor and delivery. b. registered nurse with advanced practice education. c. specially trained coach that stays with the mother during labor and delivery. d. medical doctor with advanced education in women's health.

specially trained coach that stays with the mother during labor and delivery.

A _________ ________ can now be programmed to destroy a defective gene, allowing a pure gene to replace it.

suicide gene

After a precipitous delivery, the nurse notes that the new mother is passive and only touches her newborn infant briefly with her fingertips. The nurse would do which of the following to help the woman process what has happened?

support the mother in her reaction to the newborn infant.

In speaking with a group of parents, the health-care worker informs them that in the case of a premature delivery, the fetus of 25 weeks gestation has many attributes that would help support extrauterine life. These include: (Select all that apply.) a. surfactant in the lungs. b. respiratory movements. c. a functioning central nervous system (CNS). d. smooth skin. e. subcutaneous fat.

surfactant in the lungs. respiratory movements.

A young, newly pregnant woman who led a sedentary life prior to pregnancy asks what sort of exercise would be beneficial. Helpful suggestions would be to engage in regular: a. aggressive exercise at a gym. b. jogging. c. lifting free weights. d. swimming.

swimming.


Ensembles d'études connexes

6270 - chapter 8,9,10,11 respiratory tract/COPD

View Set

MGT 499 Final Exam Chap. 9 - Corporate-level Strategy : Horizontal Integration and Vertical Integration

View Set

Pharm Exam NCLEX Cardiovascular to Maternity

View Set

PRINCIPLES of MACROECONOMICS Midterm Study set

View Set

Selecciona la respuesta correcta

View Set